Question

Question 8 4 pts You are the financial advisor of Mr. Smith. There are two stocks, A and B, available for investment E (Ra) =

0 0
Add a comment Improve this question Transcribed image text
Answer #1

Given about 2 stocks,

Standard deviation of Stock A SD(A) = 5%

Standard deviation on stock B SD(B) = 9%

Correlation between two stock Corr(A,B) = 0.5

So, Covariance between stock Cov(A,B) = SD(A)*SD(B)*Corr(A,B) = 5*9*0.5 = 22.5

For a minimum variance portfolio, weight of stock A is calculated as follow:

Wa = ((SD(B)^2) - Cov(A,B))/((SD(A)^2) + (SD(B)^2) - 2*Cov(A,B))

=> Wa = (9^2 - 22.5 )/(5^2 + 9^2 - 2*22.5) = 0.9590

So, weight of stock B = 1 - Wa = 1 - 0.9590 = 0.0410

Beta of stock A Ba = 0.8

Beta of stock B Bb = 1.2

So, beta of portfolio is weighted average beta of its assets

=> Beta of portfolio = Wa*Ba + Wb*Ba = 0.959*0.8 + 0.041*1.2 = 0.8164

So, Beta of minimum variance portfolio is 0.8164

Option A is correct.

Add a comment
Know the answer?
Add Answer to:
Question 8 4 pts You are the financial advisor of Mr. Smith. There are two stocks,...
Your Answer:

Post as a guest

Your Name:

What's your source?

Earn Coins

Coins can be redeemed for fabulous gifts.

Not the answer you're looking for? Ask your own homework help question. Our experts will answer your question WITHIN MINUTES for Free.
Similar Homework Help Questions
  • P.14 An investor holding a portfolio consisting of two stocks invests 25% of assets in Stock...

    P.14 An investor holding a portfolio consisting of two stocks invests 25% of assets in Stock A and 75% into Stock B. The return RA from Stock A has a mean of 4% and a standard deviation of A = 8%. Stock B has an expected return E(RB) = 8% with a standard deviation of ob = 12%. The portfolio return is P = 0.25RA +0.75RB. (a) Compute the expected return on the portfolio. (b) Compute the standard deviation of...

  • Question 1: You are planning about putting some money in the stock market. There are two...

    Question 1: You are planning about putting some money in the stock market. There are two stocks in your mind: stock A and stock B. The economy can either go in recession or it will boom in the coming years. Being an optimistic investor, you believe the likelihood of observing an economic boom is two times as high as observing an economic depression. You also know the following about your two stocks: State of the Economy Probability RA RB Boom...

  • Portfolio X consists of 4 stocks which are A, B, C, and D. The information pertaining...

    Portfolio X consists of 4 stocks which are A, B, C, and D. The information pertaining to the stocks, the portfolio and the market are given below: Stock Investment Beta A $25,000 0.8 B $25,000 1.2 C $25,000 Not Available D $25,000 Not Available Portfolio X $100,000 1 Expected return of the market = 10% Risk-free rate = 4% (a) Calculate the beta of Portfolio Y that is equally invested in stock A and stock B. b) Compute the beta...

  • The information regarding a portfolio consisting of two stocks is given below. Stock A E(R). 15%...

    The information regarding a portfolio consisting of two stocks is given below. Stock A E(R). 15% 17% Standard Deviation 10% 14% Stock B The correlation coefficient between Stocks A and C is zero. It can be inferred that the standard deviation of a portfolio consisting of these two stocks - Zero. Select one: O a. can not be O b. can be

  • Additional Problem 8-4 The index model has been estimated for stocks A and B with the...

    Additional Problem 8-4 The index model has been estimated for stocks A and B with the following results: RA= 0.12 +0.695RM + eA RB= 0.04 + 1.552RM+ EB om=0.355 (EA) = 0.20 O(EB) = 0.10 What is the correlation coefficient between the two stocks? (Round your answer to 4 decimal places.) Correlation coefficient

  • please answer 6. Calculating a beta coefficient for a single stock Aa Aa Suppose that the...

    please answer 6. Calculating a beta coefficient for a single stock Aa Aa Suppose that the standard deviation of returns for a single stock A is σΑ-30%, and the standard deviation of the market return is 얘-10%. If the correlation between stock A and the market is ρΑΜ-0.3, then the stock's beta is Is it reasonable to expect that the volatility of the market portfolio's future expected returns will be greater than the volatility of stock A's returns? O Yes...

  • please answer 6. Calculating a beta coefficient for a single stock Aa Aa Suppose that the...

    please answer 6. Calculating a beta coefficient for a single stock Aa Aa Suppose that the standard deviation of returns for a single stock A is σΑ-30%, and the standard deviation of the market return is 얘-10%. If the correlation between stock A and the market is ρΑΜ-0.3, then the stock's beta is Is it reasonable to expect that the volatility of the market portfolio's future expected returns will be greater than the volatility of stock A's returns? O Yes...

  • statistics 4. An investor holds a portfolio consisting of two stocks. She puts 25% of her...

    statistics 4. An investor holds a portfolio consisting of two stocks. She puts 25% of her money in Stock A and 75% into Stock B. Stock A has an expected return of Ri=8% and a standard deviation of 0,=12%. Stock B has an expected return of Rg=15% with a standard deviation of o,=22%. The portfolio return is P=0.25RA +0.75R, (a) Compute the expected return on the portfolio. (b) Compute the standard deviation of the returns on the portfolio assuming that...

  • Please show working for all parts. 1. The annual returns of two stocks are given as...

    Please show working for all parts. 1. The annual returns of two stocks are given as follows. Year Stock A Stock B 2011 -10% 21% 2012 2013 20% 5% 7% 30% 2014 -5% -3% 2015 2% -8% 2016 9% 25% (a) Estimate the expected return and volatility of each stock. (b) Estimate the covariance and correlation between two stocks. (c) Find the expected returns and volatilities of portfolios that maintain 100.6% investment in Stock A and 100(1-x)% in Stock B,...

  • Question 1. Portfolio Analysis (2 points) a) Assume the following about assets A and B: E[r]=0.1,...

    Question 1. Portfolio Analysis (2 points) a) Assume the following about assets A and B: E[r]=0.1, o =0.09. E[ra] -0.08, o; -0.04. Which one has lower absolute risk and which one has lower relative risk? (1): (2) b) Find the mean return, E[r], and variance, o. of a portfolio consisting of 70% of your total wealth invested in asset A (Wx=0.70), and 30% of your total wealth invested in asset B (w3= 0.30). The correlation between assets A and B...

ADVERTISEMENT
Free Homework Help App
Download From Google Play
Scan Your Homework
to Get Instant Free Answers
Need Online Homework Help?
Ask a Question
Get Answers For Free
Most questions answered within 3 hours.
ADVERTISEMENT
ADVERTISEMENT
ADVERTISEMENT